Difference between revisions of "2021 AIME II Problems/Problem 3"
Arnigam2007 (talk | contribs) (→Solution 2) |
MRENTHUSIASM (talk | contribs) (→Solution 2) |
||
Line 9: | Line 9: | ||
==Solution 2== | ==Solution 2== | ||
The expression <math>x_1x_2x_3 + x_2x_3x_4 + x_3x_4x_5 + x_4x_5x_1 + x_5x_1x_2</math> has cyclic symmetry. Without the loss of generality, let <math>x_1=3.</math> It follows that <math>\{x_2,x_3,x_4,x_5\}=\{1,2,4,5\}.</math> We have | The expression <math>x_1x_2x_3 + x_2x_3x_4 + x_3x_4x_5 + x_4x_5x_1 + x_5x_1x_2</math> has cyclic symmetry. Without the loss of generality, let <math>x_1=3.</math> It follows that <math>\{x_2,x_3,x_4,x_5\}=\{1,2,4,5\}.</math> We have | ||
− | # <math>x_1x_2x_3 + x_2x_3x_4 + x_3x_4x_5 + x_4x_5x_1 + x_5x_1x_2\equiv x_2x_3x_4 + x_3x_4x_5\pmod{3}</math> | + | # <math>x_1x_2x_3 + x_2x_3x_4 + x_3x_4x_5 + x_4x_5x_1 + x_5x_1x_2\equiv x_2x_3x_4 + x_3x_4x_5\pmod{3}.</math> |
# <math>x_2,x_3,x_4,x_5</math> are congruent to <math>1,2,1,2\pmod{3}</math> in some order. | # <math>x_2,x_3,x_4,x_5</math> are congruent to <math>1,2,1,2\pmod{3}</math> in some order. | ||
+ | |||
+ | We construct the following table for the case <math>x_1,</math> with all values in modulo <math>3:</math> | ||
+ | <cmath>\begin{array}{c|c|c|c|c|c} | ||
+ | \boldsymbol{x_2} & \boldsymbol{x_3} & \boldsymbol{x_4} & \boldsymbol{x_5} & \boldsymbol{x_2x_3x_4 + x_3x_4x_5} & \textbf{Valid?} \\ | ||
+ | \hline | ||
+ | & & & & & \\ [-2ex] | ||
+ | 1 & 1 & 2 & 2 & 0 & \checkmark \\ | ||
+ | 1 & 2 & 1 & 2 & 0 & \checkmark \\ | ||
+ | 1 & 2 & 2 & 1 & 2 & \\ | ||
+ | 2 & 1 & 1 & 2 & 1 & \\ | ||
+ | 2 & 1 & 2 & 1 & 0 & \checkmark \\ | ||
+ | 2 & 2 & 1 & 1 & 0 & \checkmark | ||
+ | \end{array}</cmath> | ||
<b>I am on my way. No edit please. A million thanks.</b> | <b>I am on my way. No edit please. A million thanks.</b> |
Revision as of 00:49, 23 March 2021
Problem
Find the number of permutations of numbers such that the sum of five products
Solution 1
Since is one of the numbers, a product with a in it is automatically divisible by , so WLOG , we will multiply by afterward since any of would be , after some cancelation we see that now all we need to find is the number of ways that is divisible by , since is never divisible by , now we just need to find the number of ways is divisible by , after some calculation you will see that there are ways to choose and in this way. So the desired answer is .
~ math31415926535
Solution 2
The expression has cyclic symmetry. Without the loss of generality, let It follows that We have
- are congruent to in some order.
We construct the following table for the case with all values in modulo
I am on my way. No edit please. A million thanks.
~MRENTHUSIASM
Solution 3
See also
2021 AIME II (Problems • Answer Key • Resources) | ||
Preceded by Problem 2 |
Followed by Problem 4 | |
1 • 2 • 3 • 4 • 5 • 6 • 7 • 8 • 9 • 10 • 11 • 12 • 13 • 14 • 15 | ||
All AIME Problems and Solutions |
The problems on this page are copyrighted by the Mathematical Association of America's American Mathematics Competitions.